2.Tutorium 31.3.2014

Antworten
Benutzeravatar
Mat7
Beiträge: 5
Registriert: 14.01.2014, 21:45

2.Tutorium 31.3.2014

Beitrag von Mat7 »

Hallo,

1. b) Ich habe das erste Beispiel gemacht, aber am Ende bekomme ich nur diese Beziehung, die sehr trivial aussieht:
\lim_{\tau \to +\infty} \frac{1}{\tau}( w(x,t+\tau)-w(x,t) ) = \frac{\partial}{\partial t}w(x,t)

Ist es korrekt?
Wenn ihr auch es gefunden habt, mochte ich wissen, wie kann man es interpretieren???

3. Beim 3. Beispiel wollte ich <I(\tau)I(\tau+t)> berechnen, aber dann hab ich nicht gewusst, wie man zwei Dirac-Deltafunktionen multiplizieren konnte.

Danke!

gustaf2000
Beiträge: 18
Registriert: 27.05.2009, 15:06

Re: 2.Tutorium 31.3.2014

Beitrag von gustaf2000 »

Hallo,

zu 1: wenn man bei a) die Wahrscheinlichkeit mit 0,5 einsetzt, bleiben einem immer noch die Summen. Und die hat man dann auch noch im Differenzenquotienten - oder habe ich da eine Vereinfachung übersehen?

3: Würde ich nicht so machen. Das klingt nach harmonischer Analyse (Plenum), da ja nur die spektrale Dichte gefragt ist, und nicht I(t). Aber was auf den ersten Blick einfach aussieht, ist auch etwas heikel. Die FT der Deltafunktion wäre 1, und dann I(w)= q.N
Laut Skript ist aber v(w) ....exp(+iwt) statt exp(-iwt) und dann hat man es schwer?

2: für die Geschwindigkeitskorrelation komme ich auf einen riesigen Ausdruck, weil da drei-teilige Terme miteinander multipliziert werden und viel stehenbleibt.

4: Ist dem Plenum sehr ähnlich, auch die Ergebnisse sind überschaubar.

Benutzeravatar
Mat7
Beiträge: 5
Registriert: 14.01.2014, 21:45

Re: 2.Tutorium 31.3.2014

Beitrag von Mat7 »

Hallo,

1. Die Summen bleiben nicht mehr, weil der Random-Walker nur einen Schritt nach links oder rechts gehen darf. d.h.: m ist nicht mehr von -inf bis +inf, sondern m=n-1 oder n+1.(mit gleichen P=0.5)

3. I(w)=qN? das stimmt nicht.I(\omega)=\int_{-\infty}^{+\infty} \sum_{n=1}^{N}\delta(t-t_n)e^{-2\pi i \omega t}=\sum_{n=1}^{N}\int_{-\infty}^{+\infty} \delta(t-t_n)e^{-2\pi i \omega t}=\sum_{n=1}^{N}e^{-2\pi i \omega t_n}

was sollte ich danach machen?

Benutzeravatar
Rumte
Beiträge: 121
Registriert: 22.03.2007, 11:51
Wohnort: Wien

Re: 2.Tutorium 31.3.2014

Beitrag von Rumte »

Hallo,

kann mir jemand erklären wieso ich beim Autokorrelationsintegral eine Fallunterscheidung machen muss?

Danke.

John Doe
Beiträge: 10
Registriert: 26.01.2007, 09:40

Re: 2.Tutorium 31.3.2014

Beitrag von John Doe »

Hi,

ad 1. Wenn man in der Gleichung für den Differenzialquotienten die gegebenen Substitutionen vornimmt, eine Taylorentwicklung nach Ort und Zeit macht (2. Ordnung nicht vergessen) und dann den Limes bildet, sollte man auf die Diffusionsgleichung kommen.

gustaf2000
Beiträge: 18
Registriert: 27.05.2009, 15:06

Re: 2.Tutorium 31.3.2014

Beitrag von gustaf2000 »

Hallo,

zu Mat7 / John Doe
zu 1
Danke für die Summen; dein Ergebnis aus dem ersten Beitrag ist die Definition des Differenzenquotienten. Hätte es vielleicht Vorteile, wenn man die Übergangswahrscheinlichkeit nicht mit 0,5 einsetzt, sondern wie John Doe meint, als Reihe entwickelt. Hier

http://wwwcp.tphys.uni-heidelberg.de/st ... h-kap3.pdf

hat man nur nach dem Ort entwickelt, aber sattelt man da nicht das Pferd von hinten auf? Wenn man das nicht macht und nur brav die zwei verbleibenden Terme der Summe in die Gleichung für den Differenzenquotienten einsetzt, kommt man beim Limes zu 0/0. Nicht sinnvoll.


zu 3
Warum steht bei dir 2.pi im Exponenten? Die Formel für die Fouriertranformation in den Frequenzraum aus dem Mathebuch hat nur exp(-iwt) und mit der Deltafunktion als zu transformierende Funktion ist das ganze Integral per Definition 1. Die Argumentation damit man Summe und Integral vertauschen darf, ist nicht so locker, da die Deltafunktion nicht stetig und nicht begrenzt ist. Aber gehen wir mal davon aus, dass man es darf, hätte man N mal 1 zu summieren, und damit I(w)=q.N
Damit wäre aber die spektrale Dichte S(w) = 0?

imnosheldon
Beiträge: 31
Registriert: 14.03.2012, 16:07

Re: 2.Tutorium 31.3.2014

Beitrag von imnosheldon »

mag wer seine lösungswege hochladen? komm bei keinem der beispiele auf einen grünen zweig... :roll:

vitarmind
Beiträge: 11
Registriert: 13.05.2009, 22:58

Re: 2.Tutorium 31.3.2014

Beitrag von vitarmind »

Rumte hat geschrieben:Hallo,

kann mir jemand erklären wieso ich beim Autokorrelationsintegral eine Fallunterscheidung machen muss?
Das kommt von der zeitlichen deltakorrelation einer stochastischen kraft. wenn du die integrieren willst, erhältst du nur etwas, wenn das argument null wird, also t'=t'' sein kann. das ist nicht unbedingt der fall. aber es ist der fall, wenn die zeitvariable, über die integriert wird, eine höhere obere grenze hat als die andere, über die du später integrierst.

hab dafür auch mehrmals anleitung gebraucht ;)

gustaf2000
Beiträge: 18
Registriert: 27.05.2009, 15:06

Re: 2.Tutorium 31.3.2014

Beitrag von gustaf2000 »

Hier meine Notizen zu 2 und 4.

Verbesserungsvorschläge gerne willkommen!

Habt ihr Vorschläge zu 1 und 3?
Bsp. 2.pdf
Bsp. 4.pdf
Du hast keine ausreichende Berechtigung, um die Dateianhänge dieses Beitrags anzusehen.

John Doe
Beiträge: 10
Registriert: 26.01.2007, 09:40

Re: 2.Tutorium 31.3.2014

Beitrag von John Doe »

gustaf2000 hat geschrieben:Hallo,

zu Mat7 / John Doe
zu 1
Danke für die Summen; dein Ergebnis aus dem ersten Beitrag ist die Definition des Differenzenquotienten. Hätte es vielleicht Vorteile, wenn man die Übergangswahrscheinlichkeit nicht mit 0,5 einsetzt, sondern wie John Doe meint, als Reihe entwickelt.
Nicht die Übergangswahrscheinlichkeiten (die würde ich mit 0,5 angesetzt lassen) sollten entwickelt werden sondern die w_1. Dann bleibt einem neben der ersten Zeitableitung wegen \frac{\Delta^2}{\tau} = const auch eine zweite Ortsableitung übrig. Die Gleichung von der man wegstartet erhält man durch ersetzen von w_1 im Differenzialquotient wie im Unterpunkt 1a bestimmt.

John Doe
Beiträge: 10
Registriert: 26.01.2007, 09:40

Re: 2.Tutorium 31.3.2014

Beitrag von John Doe »

gustaf2000 hat geschrieben:Hier meine Notizen zu 2 und 4.

Verbesserungsvorschläge gerne willkommen!
Thx, hab mir gerade das 4er angeschaut, bei der Varianz müsste sowas wie 1 - exp(-2t) rauskommen.

Antworten

Zurück zu „Statistische Physik II“